Proof that is irrational

Results: 99



#Item
1

A SIMPLE PROOF THAT TT IS IRRATIONAL IVAN NIVEN Let 7T = a/6, the quotient of positive integers. We define the polynomials xn(a — bx)n

Add to Reading List

Source URL: www.ams.org

- Date: 2010-01-14 13:38:21
    2Mathematics / Mathematical analysis / Algebra / Exponentials / Sine / Trigonometric functions / Exponentiation / Complex number / Proof that  is irrational / Pythagorean triple

    International Mathematical Olympiad Preliminary Selection ContestHong Kong Answers and Suggested Solutions Answers:

    Add to Reading List

    Source URL: web.hku.hk

    Language: English - Date: 2013-03-15 02:41:05
    3Mathematical analysis / Special functions / Mathematics / Trigonometry / Orthogonal polynomials / Ratios / Trigonometric functions / Sine / Chebyshev polynomials / Proof that  is irrational

    2003 Mathematics Advanced Higher Finalised Marking Instructions 2003 Mathematics Advanced Higher – Section A

    Add to Reading List

    Source URL: www.sqa.org.uk

    Language: English - Date: 2003-09-11 10:36:11
    4Mathematical analysis / Mathematics / Special functions / Exponentials / Analytic functions / Ratios / Trigonometric functions / Trigonometry / Sine / Parametric equation / Proof that  is irrational

    $SPAD/src/input kamke0.input Timothy Daly December 30, 2008 Abstract This is the first 50 of the Kamke test suite as published by E. S. ChebTerrab[1]. They have been rewritten using Axiom syntax. Where possible we show t

    Add to Reading List

    Source URL: axiom-developer.org

    Language: English - Date: 2008-12-30 18:00:15
    5Mathematical analysis / Mathematics / Approximation theory / Chebyshev polynomials / Normal distribution / Exponentiation / Bernoulli polynomials / Proof that  is irrational

    The ratio between the Toeplitz and the unstructured condition number Siegfried M. Rump and H. Sekigawa Abstract. Recently it was shown that the ratio between the normwise Toeplitz structured condition number of a linear

    Add to Reading List

    Source URL: www.ti3.tu-harburg.de

    Language: English - Date: 2008-01-09 07:58:25
    6Mathematics / Mathematical analysis / Elementary algebra / Trigonometry / Inverse trigonometric functions / Ratios / Nth root / Factorization / E / Transcendental number / Proof that  is irrational

    International Mathematical Olympiad Preliminary Selection Contest 2012 — Hong Kong Outline of Solutions Answers:

    Add to Reading List

    Source URL: web.hku.hk

    Language: English - Date: 2013-03-15 02:35:42
    7Functions and mappings / Fourier analysis / Calculus / Derivative / Differential calculus / Rates / Periodic function / Intermediate value theorem / Multiplicative inverse / Limit of a function / Proof that  is irrational

    CLASS QUIZ: OCTOBER 12: DERIVATIVES MATH 152, SECTION 55 (VIPUL NAIK) Your name (print clearly in capital letters): Write your answer in the space provided. In the space below, you can explain your work if you want (this

    Add to Reading List

    Source URL: files.vipulnaik.com

    Language: English - Date: 2016-08-13 11:33:29
    8Integral calculus / Functions and mappings / Integration by parts / Antiderivative / Integral / Derivative / Rational function / Continuous function / Lebesgue integration / Differentiation rules / Proof that  is irrational

    TAKE-HOME CLASS QUIZ: DUE WEDNESDAY FEBRUARY 20: INTEGRATION TECHNIQUES (ONE VARIABLE) MATH 195, SECTION 59 (VIPUL NAIK) Your name (print clearly in capital letters): In the questions below, we say that a function is exp

    Add to Reading List

    Source URL: files.vipulnaik.com

    Language: English - Date: 2016-08-13 11:33:29
    9Mathematical analysis / Mathematics / Analysis / Operator theory / Algebra of random variables / Covariance / Covariance and correlation / Variance / Monotone convergence theorem / Pi / Harmonic analysis / Proof that  is irrational

    Telling cause from effect in deterministic linear dynamical systems Supplementary Material We have prepared an appendix to address the proofs for Proposition 3, Theorems 1 and 2 which we provide in the following sections

    Add to Reading List

    Source URL: jmlr.org

    Language: English - Date: 2015-09-16 19:38:47
    10

    NON-REGULARITY OF bα + logk nc ERIC S. ROWLAND Abstract. This paper presents a new proof that if kα is irrational then the sequence {bα+logk nc}n≥1 is not k-regular. Unlike previous proofs, the methods used do not r

    Add to Reading List

    Source URL: thales.math.uqam.ca

    Language: English - Date: 2009-11-21 15:03:08
      UPDATE